PSAT Writing and Language Question 393: Answer and Explanation

Question: 393

  • A. NO CHANGE
  • B. As a result of
  • C. In addition to
  • D. Instead of

Correct Answer: D

Explanation:

(D) The previous sentence establishes that the narrator has lost joy in reading. The current sentence serves to explain how this shift in attitude came about—namely, rather than reading recreationally, the narrator was required to read certain texts. Choice (D) is the only option that shows this contrast. Choices (A) and (B) both show cause and effect, and choice (C) indicates a list.

Test Information

More Tests

    All content of site and practice tests © 2022 Jack.
    Quick View

    PSAT Practice Tests

    More Information